Essential University Physics: Volume 1 (4th Edition)

Published by Pearson
ISBN 10: 0-134-98855-8
ISBN 13: 978-0-13498-855-9

Chapter 13 - Section 13.3 - Applications of Simple Harmonic Motion - Example - Page 238: 13.4

Answer

1.6 seconds

Work Step by Step

We find: $T = \frac{2\pi}{\omega}=2\pi \times \sqrt{\frac{I}{mgL}} = 2\pi \times \sqrt{\frac{\frac{1}{3}m(2L^2)}{mgL}} = 2\pi \times \sqrt{\frac{4L}{3g}}$ Plugging in the values gives an answer of $\fbox{1.6 seconds}.$
Update this answer!

You can help us out by revising, improving and updating this answer.

Update this answer

After you claim an answer you’ll have 24 hours to send in a draft. An editor will review the submission and either publish your submission or provide feedback.